how to prove limit exists or not?Limit $fracx^2yx^4+y^2$ is found using polar coordinates but it is not supposed to exist.Compute the limit of $frac + e^right)sqrtx^2 + y^2$ when $(x,y)to (0,0)$Calculate double limit of $x^2sinfrac1xy$Rational Multivariable limitIs polar coordinates enough to prove that a limit existsProving the limit exists or not for multivariable functions.Basic Multivariable LimitTwo-variable limit of $lim_(x,y)to(0,0)fracsin(x^4+y^4)x^2+y^2$finf if the limit exists for a 2 variable functionsSolving a sequence limit with floor

Do I really need to have a scientific explanation for my premise?

From an axiomatic set theoric approach why can we take uncountable unions?

Can the alpha, lambda values of a glmnet object output determine whether ridge or Lasso?

The meaning of ‘otherwise’

Would an aboleth's Phantasmal Force lair action be affected by Counterspell, Dispel Magic, and/or Slow?

Why do we say ‘pairwise disjoint’, rather than ‘disjoint’?

Virginia employer terminated employee and wants signing bonus returned

Professor forcing me to attend a conference, I can't afford even with 50% funding

Signed and unsigned numbers

How does Ehrenfest's theorem apply to the quantum harmonic oscillator?

Are all players supposed to be able to see each others' character sheets?

Are small insurances worth it?

Why is a very small peak with larger m/z not considered to be the molecular ion?

Plausibility of Mushroom Buildings

Expressing logarithmic equations without logs

Finitely many repeated replacements

Is it a Cyclops number? "Nobody" knows!

In the late 1940’s to early 1950’s what technology was available that could melt ice?

Having the player face themselves after the mid-game

Why does Solve lock up when trying to solve the quadratic equation with large integers?

Shifting between bemols and diesis in the key signature

How to write a chaotic neutral protagonist and prevent my readers from thinking they are evil?

MySQL importing CSV files really slow

NASA's RS-25 Engines



how to prove limit exists or not?


Limit $fracx^2yx^4+y^2$ is found using polar coordinates but it is not supposed to exist.Compute the limit of $fraclog left(sqrtx^2 + y^2$ when $(x,y)to (0,0)$Calculate double limit of $x^2sinfrac1xy$Rational Multivariable limitIs polar coordinates enough to prove that a limit existsProving the limit exists or not for multivariable functions.Basic Multivariable LimitTwo-variable limit of $lim_(x,y)to(0,0)fracsin(x^4+y^4)x^2+y^2$finf if the limit exists for a 2 variable functionsSolving a sequence limit with floor













1












$begingroup$


I am having hard time understanding what is the correct way to prove whether the limit exists or not for two variable functions of those two functions.



I don't know how to solve the first limit.
In the second limit I've tried to use $$x=rcos theta, y=rsin theta$$
but it didn't help me. I tried to use Squeeze theorem and got stuck.



$$lim_(x,y)to(0,0)fracsin(x^n+y^n)x^n-1+y^n-1$$



$$lim_(x,y)to(0,0)fracx^2cdot y^5x^3+y^9$$










share|cite|improve this question









New contributor




Rotemi K is a new contributor to this site. Take care in asking for clarification, commenting, and answering.
Check out our Code of Conduct.







$endgroup$







  • 1




    $begingroup$
    Welcome to MSE. Your question is phrased as an isolated problem, without any further information or context. This does not match many users' quality standards, so it may attract downvotes, or be put on hold. To prevent that, please edit the question. This will help you recognise and resolve the issues. Concretely: please provide context, and include your work and thoughts on the problem. These changes can help in formulating more appropriate answers.
    $endgroup$
    – José Carlos Santos
    yesterday










  • $begingroup$
    Hi, thanks for the comment. what should I add/edit in the post?
    $endgroup$
    – Rotemi K
    yesterday










  • $begingroup$
    You could edit your post, adding to it what you have done thus far.
    $endgroup$
    – José Carlos Santos
    yesterday










  • $begingroup$
    Ok, thank you- edited.
    $endgroup$
    – Rotemi K
    yesterday















1












$begingroup$


I am having hard time understanding what is the correct way to prove whether the limit exists or not for two variable functions of those two functions.



I don't know how to solve the first limit.
In the second limit I've tried to use $$x=rcos theta, y=rsin theta$$
but it didn't help me. I tried to use Squeeze theorem and got stuck.



$$lim_(x,y)to(0,0)fracsin(x^n+y^n)x^n-1+y^n-1$$



$$lim_(x,y)to(0,0)fracx^2cdot y^5x^3+y^9$$










share|cite|improve this question









New contributor




Rotemi K is a new contributor to this site. Take care in asking for clarification, commenting, and answering.
Check out our Code of Conduct.







$endgroup$







  • 1




    $begingroup$
    Welcome to MSE. Your question is phrased as an isolated problem, without any further information or context. This does not match many users' quality standards, so it may attract downvotes, or be put on hold. To prevent that, please edit the question. This will help you recognise and resolve the issues. Concretely: please provide context, and include your work and thoughts on the problem. These changes can help in formulating more appropriate answers.
    $endgroup$
    – José Carlos Santos
    yesterday










  • $begingroup$
    Hi, thanks for the comment. what should I add/edit in the post?
    $endgroup$
    – Rotemi K
    yesterday










  • $begingroup$
    You could edit your post, adding to it what you have done thus far.
    $endgroup$
    – José Carlos Santos
    yesterday










  • $begingroup$
    Ok, thank you- edited.
    $endgroup$
    – Rotemi K
    yesterday













1












1








1


0



$begingroup$


I am having hard time understanding what is the correct way to prove whether the limit exists or not for two variable functions of those two functions.



I don't know how to solve the first limit.
In the second limit I've tried to use $$x=rcos theta, y=rsin theta$$
but it didn't help me. I tried to use Squeeze theorem and got stuck.



$$lim_(x,y)to(0,0)fracsin(x^n+y^n)x^n-1+y^n-1$$



$$lim_(x,y)to(0,0)fracx^2cdot y^5x^3+y^9$$










share|cite|improve this question









New contributor




Rotemi K is a new contributor to this site. Take care in asking for clarification, commenting, and answering.
Check out our Code of Conduct.







$endgroup$




I am having hard time understanding what is the correct way to prove whether the limit exists or not for two variable functions of those two functions.



I don't know how to solve the first limit.
In the second limit I've tried to use $$x=rcos theta, y=rsin theta$$
but it didn't help me. I tried to use Squeeze theorem and got stuck.



$$lim_(x,y)to(0,0)fracsin(x^n+y^n)x^n-1+y^n-1$$



$$lim_(x,y)to(0,0)fracx^2cdot y^5x^3+y^9$$







calculus limits






share|cite|improve this question









New contributor




Rotemi K is a new contributor to this site. Take care in asking for clarification, commenting, and answering.
Check out our Code of Conduct.











share|cite|improve this question









New contributor




Rotemi K is a new contributor to this site. Take care in asking for clarification, commenting, and answering.
Check out our Code of Conduct.









share|cite|improve this question




share|cite|improve this question








edited yesterday







Rotemi K













New contributor




Rotemi K is a new contributor to this site. Take care in asking for clarification, commenting, and answering.
Check out our Code of Conduct.









asked yesterday









Rotemi KRotemi K

112




112




New contributor




Rotemi K is a new contributor to this site. Take care in asking for clarification, commenting, and answering.
Check out our Code of Conduct.





New contributor





Rotemi K is a new contributor to this site. Take care in asking for clarification, commenting, and answering.
Check out our Code of Conduct.






Rotemi K is a new contributor to this site. Take care in asking for clarification, commenting, and answering.
Check out our Code of Conduct.







  • 1




    $begingroup$
    Welcome to MSE. Your question is phrased as an isolated problem, without any further information or context. This does not match many users' quality standards, so it may attract downvotes, or be put on hold. To prevent that, please edit the question. This will help you recognise and resolve the issues. Concretely: please provide context, and include your work and thoughts on the problem. These changes can help in formulating more appropriate answers.
    $endgroup$
    – José Carlos Santos
    yesterday










  • $begingroup$
    Hi, thanks for the comment. what should I add/edit in the post?
    $endgroup$
    – Rotemi K
    yesterday










  • $begingroup$
    You could edit your post, adding to it what you have done thus far.
    $endgroup$
    – José Carlos Santos
    yesterday










  • $begingroup$
    Ok, thank you- edited.
    $endgroup$
    – Rotemi K
    yesterday












  • 1




    $begingroup$
    Welcome to MSE. Your question is phrased as an isolated problem, without any further information or context. This does not match many users' quality standards, so it may attract downvotes, or be put on hold. To prevent that, please edit the question. This will help you recognise and resolve the issues. Concretely: please provide context, and include your work and thoughts on the problem. These changes can help in formulating more appropriate answers.
    $endgroup$
    – José Carlos Santos
    yesterday










  • $begingroup$
    Hi, thanks for the comment. what should I add/edit in the post?
    $endgroup$
    – Rotemi K
    yesterday










  • $begingroup$
    You could edit your post, adding to it what you have done thus far.
    $endgroup$
    – José Carlos Santos
    yesterday










  • $begingroup$
    Ok, thank you- edited.
    $endgroup$
    – Rotemi K
    yesterday







1




1




$begingroup$
Welcome to MSE. Your question is phrased as an isolated problem, without any further information or context. This does not match many users' quality standards, so it may attract downvotes, or be put on hold. To prevent that, please edit the question. This will help you recognise and resolve the issues. Concretely: please provide context, and include your work and thoughts on the problem. These changes can help in formulating more appropriate answers.
$endgroup$
– José Carlos Santos
yesterday




$begingroup$
Welcome to MSE. Your question is phrased as an isolated problem, without any further information or context. This does not match many users' quality standards, so it may attract downvotes, or be put on hold. To prevent that, please edit the question. This will help you recognise and resolve the issues. Concretely: please provide context, and include your work and thoughts on the problem. These changes can help in formulating more appropriate answers.
$endgroup$
– José Carlos Santos
yesterday












$begingroup$
Hi, thanks for the comment. what should I add/edit in the post?
$endgroup$
– Rotemi K
yesterday




$begingroup$
Hi, thanks for the comment. what should I add/edit in the post?
$endgroup$
– Rotemi K
yesterday












$begingroup$
You could edit your post, adding to it what you have done thus far.
$endgroup$
– José Carlos Santos
yesterday




$begingroup$
You could edit your post, adding to it what you have done thus far.
$endgroup$
– José Carlos Santos
yesterday












$begingroup$
Ok, thank you- edited.
$endgroup$
– Rotemi K
yesterday




$begingroup$
Ok, thank you- edited.
$endgroup$
– Rotemi K
yesterday










0






active

oldest

votes











Your Answer





StackExchange.ifUsing("editor", function ()
return StackExchange.using("mathjaxEditing", function ()
StackExchange.MarkdownEditor.creationCallbacks.add(function (editor, postfix)
StackExchange.mathjaxEditing.prepareWmdForMathJax(editor, postfix, [["$", "$"], ["\\(","\\)"]]);
);
);
, "mathjax-editing");

StackExchange.ready(function()
var channelOptions =
tags: "".split(" "),
id: "69"
;
initTagRenderer("".split(" "), "".split(" "), channelOptions);

StackExchange.using("externalEditor", function()
// Have to fire editor after snippets, if snippets enabled
if (StackExchange.settings.snippets.snippetsEnabled)
StackExchange.using("snippets", function()
createEditor();
);

else
createEditor();

);

function createEditor()
StackExchange.prepareEditor(
heartbeatType: 'answer',
autoActivateHeartbeat: false,
convertImagesToLinks: true,
noModals: true,
showLowRepImageUploadWarning: true,
reputationToPostImages: 10,
bindNavPrevention: true,
postfix: "",
imageUploader:
brandingHtml: "Powered by u003ca class="icon-imgur-white" href="https://imgur.com/"u003eu003c/au003e",
contentPolicyHtml: "User contributions licensed under u003ca href="https://creativecommons.org/licenses/by-sa/3.0/"u003ecc by-sa 3.0 with attribution requiredu003c/au003e u003ca href="https://stackoverflow.com/legal/content-policy"u003e(content policy)u003c/au003e",
allowUrls: true
,
noCode: true, onDemand: true,
discardSelector: ".discard-answer"
,immediatelyShowMarkdownHelp:true
);



);






Rotemi K is a new contributor. Be nice, and check out our Code of Conduct.









draft saved

draft discarded


















StackExchange.ready(
function ()
StackExchange.openid.initPostLogin('.new-post-login', 'https%3a%2f%2fmath.stackexchange.com%2fquestions%2f3140955%2fhow-to-prove-limit-exists-or-not%23new-answer', 'question_page');

);

Post as a guest















Required, but never shown

























0






active

oldest

votes








0






active

oldest

votes









active

oldest

votes






active

oldest

votes








Rotemi K is a new contributor. Be nice, and check out our Code of Conduct.









draft saved

draft discarded


















Rotemi K is a new contributor. Be nice, and check out our Code of Conduct.












Rotemi K is a new contributor. Be nice, and check out our Code of Conduct.











Rotemi K is a new contributor. Be nice, and check out our Code of Conduct.














Thanks for contributing an answer to Mathematics Stack Exchange!


  • Please be sure to answer the question. Provide details and share your research!

But avoid


  • Asking for help, clarification, or responding to other answers.

  • Making statements based on opinion; back them up with references or personal experience.

Use MathJax to format equations. MathJax reference.


To learn more, see our tips on writing great answers.




draft saved


draft discarded














StackExchange.ready(
function ()
StackExchange.openid.initPostLogin('.new-post-login', 'https%3a%2f%2fmath.stackexchange.com%2fquestions%2f3140955%2fhow-to-prove-limit-exists-or-not%23new-answer', 'question_page');

);

Post as a guest















Required, but never shown





















































Required, but never shown














Required, but never shown












Required, but never shown







Required, but never shown

































Required, but never shown














Required, but never shown












Required, but never shown







Required, but never shown







Popular posts from this blog

Lowndes Grove History Architecture References Navigation menu32°48′6″N 79°57′58″W / 32.80167°N 79.96611°W / 32.80167; -79.9661132°48′6″N 79°57′58″W / 32.80167°N 79.96611°W / 32.80167; -79.9661178002500"National Register Information System"Historic houses of South Carolina"Lowndes Grove""+32° 48' 6.00", −79° 57' 58.00""Lowndes Grove, Charleston County (260 St. Margaret St., Charleston)""Lowndes Grove"The Charleston ExpositionIt Happened in South Carolina"Lowndes Grove (House), Saint Margaret Street & Sixth Avenue, Charleston, Charleston County, SC(Photographs)"Plantations of the Carolina Low Countrye

random experiment with two different functions on unit interval Announcing the arrival of Valued Associate #679: Cesar Manara Planned maintenance scheduled April 23, 2019 at 00:00UTC (8:00pm US/Eastern)Random variable and probability space notionsRandom Walk with EdgesFinding functions where the increase over a random interval is Poisson distributedNumber of days until dayCan an observed event in fact be of zero probability?Unit random processmodels of coins and uniform distributionHow to get the number of successes given $n$ trials , probability $P$ and a random variable $X$Absorbing Markov chain in a computer. Is “almost every” turned into always convergence in computer executions?Stopped random walk is not uniformly integrable

How should I support this large drywall patch? Planned maintenance scheduled April 23, 2019 at 00:00UTC (8:00pm US/Eastern) Announcing the arrival of Valued Associate #679: Cesar Manara Unicorn Meta Zoo #1: Why another podcast?How do I cover large gaps in drywall?How do I keep drywall around a patch from crumbling?Can I glue a second layer of drywall?How to patch long strip on drywall?Large drywall patch: how to avoid bulging seams?Drywall Mesh Patch vs. Bulge? To remove or not to remove?How to fix this drywall job?Prep drywall before backsplashWhat's the best way to fix this horrible drywall patch job?Drywall patching using 3M Patch Plus Primer